a book store bought 160 copies of a book from the publisher for $4,000. If the store gives away 2 books, how much money will it lose?

Answers

Answer 1
Answer: 4000 divide by 160 to get the cost of each book 
4000 / 160 = 25
So that's £25 per book, multiply that by two because they gave two away which is £50 so they lost £50
Answer 2
Answer: we think like each other i was about to put that

Related Questions

given triangle ABC has segment BC congruent to segment CA. Identify this type of triangle and other congruent parts
Would you rather....
What is the common difference between the elements of the arithmetic sequence below?–18, –22.5, –27, –31.5, –36
Two cars start from the same place driving at the same rate, one heading north and the other heading east. After two hours the car heading north has traveled 150 miles and the car heading east has traveled 100 miles. How far apart are the cars at this time?
Which expressions are equivalent to 2^5.2^4? Check all that apply.2^202.2^92^10 • 2²2^-2 2^11(2.2.2.2.2).(2.2.2.2)

Help meh....................​

Answers

Answer:

The Exception: Negative Numbers. There is one very important exception to the rule that multiplying or dividing an inequality is the same as multiplying or dividing an equation. Whenever you multiply or divide an inequality by a negative number, you must flip the inequality sign.

Step-by-step explanation:

tim makes 80 gallons of paint by mixing 48 gallons gray paint with 32 gallons of white paint. what part is gray paint?

Answers

So you divide 48 by a 100 and you get 0.48

Answer: 38.4

Step-by-step explanation: 48 is 38.4 percent of the 80. You can find this out by calculating for percentage

Find the mode and median of the data. A. mode = 41 median = 72 B. mode = 72 median = 94 C. mode = 94 median = 94 D. mode = 94 median = 72

Answers

mode means the number that apperss the most
median is the number in the middle when you arrange them in numerical order


ok so
you take the unbers fom the left most collumn and pair it with each number in the right side of the line, the left collumn is the tens place and the right hand side is the ones

so the th enumbers are

32,41,41,50,56,57,62,70,70,72,72,72,73,74,91,92,94,94,94,94


mode=most common number
94 appears 4 times which is most
mode=94

median is number in very middle number
20 terms
72 is in middle


mode=94
72=median

D is answer

Answer:

D

Step-by-step explanation:

You take the numbers fom the left most collumn and pair it with each number in the right side of the line, the left collumn is the tens place and the right hand side is the ones.

So the the numbers are:

32,41,41,50,56,57,62,70,70,72,72,72,73,74,91,92,94,94,94,94

Mode = Most common number

94 appears 4 times which is most

Mode=94

Median is number in very middle number

20 terms

72 is in middle

Mode = 94

Median = 72

What is |58| in absolute value?

Answers

"So in practice "absolute value" means to remove any negative sign in front of a number, and to think of all numbers as positive (or zero)."

so your answer is { 58 } :)
The answer is 58 because there is no negative sign on the outside. For instance, -|58| this would be -58 since the negative sign is on the outside.

How much does 16 go into 75

Answers

16 goes into 75 4 times, with a remainder of 11.

75/16 = 4.6875
4 x 16 = 64
75 - 64 = 11

:D

Need help pls
we use a+b+c equations barely today HELP PLS IM LOST

Answers

The answer:  59° .
______________________________________________
 
Explanation:
______________________________________________
 a + b +  c = 180 .

(in which, "a" , "b", and "c" represent the measurements (in degrees" of each of the THREE (3) angles in a triangle; which always add up to "180 degrees");
______________
Given "a = 65 degrees";  "b = 56 degrees"; Find "c" (the measurement of the "unknown" angle.).
____________________________________________
 a + b + c = 180 ;

65 + 56 + c = 180 ;  Solve for "c" .
__________________________________

  180 - (65 + 56)  = c  ;
___________________________

Note:  65 + 56 = 121 ;
_____________________________
So:  180 - (65 + 56)  = 180 - 121= 59 = c ;  ↔  c = 59 .
___________________________________________ 
The answer:  59° .
_______________________________